Math Olympiad 3^m-2^m=65 | Math Olympiad Problems | Algebra

Sdílet
Vložit
  • čas přidán 28. 06. 2024
  • In solving this math Olympiad problem, 3^m-2^m=65, Jakes uses a very unique approach to handle this exponential math challenge with easy.
    Watch the full detail in this video clip without skipping any parts of this video.
    Share this wonderful video with family members, friends and loved ones who need this knowledge using the video link below here.
    • Math Olympiad 3^m-2^m=...
    Also, I want to share with you how I have be able to use Promoterkit for free for some time now to generate more views and subscribers in the few weeks. You can clink the below to access it.
    promoterkit.com/#youtubetools
    Gain access to all videos on this channel using the link below:
    / @onlinemathstv
    Love you all 💕💖💖💖
    #matholympiad #maths #algebra

Komentáře • 3K

  • @tiehoteele874
    @tiehoteele874 Před 6 měsíci +57

    The comment section is polluted by critics but i learned some tricks from this video.. EXCELLENT

    • @onlineMathsTV
      @onlineMathsTV  Před 6 měsíci +7

      Wow!!! Thanks and we are glad you gained some values from this video tutorial sir.

    • @thefireyphoenix
      @thefireyphoenix Před 2 měsíci +1

      it isnt critism.. in every maths channel you will see people doing that.. they do it for alternate solutions and writing down their own way

    • @tiehoteele874
      @tiehoteele874 Před měsícem +1

      @@thefireyphoenix this video wasn't made for them

    • @Theuserisadick
      @Theuserisadick Před měsícem

      Polluted by critics??
      You're definitely not a student of maths

    • @najmulhussainlaskar7118
      @najmulhussainlaskar7118 Před 15 dny +1

      I learnt something new from this video... And I wanna say something to dear sir please do you job not listen other criticism... Love you sor i am from India ( Bengali) i am also a Maths teacher 🧡🤍💚

  • @mitahaubica6498
    @mitahaubica6498 Před 9 měsíci +114

    I immediately saw that 65 can be decomposed as 81-16, and conveniently 81 is 3^4 and 16 is 2^4, so matching coefficients suggests m is 4.

    • @BrightonMutero
      @BrightonMutero Před 7 měsíci +2

      You are a genius

    • @nicadi2005
      @nicadi2005 Před 7 měsíci +12

      @mitahaubica6498 "I immediately saw that 65 can be decomposed as 81-16" - That's not decomposition... You can find an infinity of pairs of numbers that have their difference equal to 65.
      The fact that you selected one such pair that also happens to be powers of the respective bases in the original problem merely indicates you have approached solving this by trial and error...
      *The question would be whether you can do better than finding the solution that way...*

    • @counterpoint9260
      @counterpoint9260 Před 6 měsíci +2

      that is not the right method..may work here but not al the time

    • @davidmajor1508
      @davidmajor1508 Před 4 měsíci +3

      You just got lucky. It was pure luck that you used the right numbers to subtract, and that m is an integer in this case.

    • @danielvazquez6301
      @danielvazquez6301 Před 2 měsíci +5

      Always try some values of m to see the behaviour.
      m=0 or 1 or 3 or 4... I've found the solution!
      Obviously it is not an Olympiad problem.

  • @danieldavies1829
    @danieldavies1829 Před 9 měsíci +142

    I worked it out a bit different. My solution was simply determine what of 3 exponent would get me a number greater than 65 that would be an odd number (3^4). I then subtracted that 65 from that number (81) and I got 16 which is 2^4.
    In other words you can rewrite the equation in this instance as
    3^m - 2^m=65
    3^m - 65 = 2^m
    The first exponent of 3 which results in a number greater than 65 is 4
    so 3^4 = 81
    81-65 = 2^m
    16 = 2^m
    16 can be written as 2^4
    m=4

    • @Xhopp3r
      @Xhopp3r Před 9 měsíci +5

      That's exactly what I did.

    • @Smith_14
      @Smith_14 Před 9 měsíci +6

      m can be a negative number?

    • @leishajuneja2994
      @leishajuneja2994 Před 9 měsíci +5

      You cant take any value of 'm'.consider the question is same but with a very large value(instead of 65),probably in crores,it wout take an eternity to reach that number

    • @leoosu
      @leoosu Před 9 měsíci +3

      I did the same😂

    • @DownhillAllTheWay
      @DownhillAllTheWay Před 9 měsíci +8

      You're assuming that m is an integer?

  • @f-s406
    @f-s406 Před 9 měsíci +95

    I got ‘m=4’ by mental arithmetic. Because 3^m > 65 and 2^m < 3^m, that’s necessary.
    The value '65' determines that the value range of m must be less than 5 and greater than 0. When m is a positive integer, test the m=5, 4, 3, 2, 1 and finally get m=4.

    • @CBSE24
      @CBSE24 Před 8 měsíci +2

      I too

    • @tuyu6404
      @tuyu6404 Před 8 měsíci +1

      Since we don't participate in the Olympics, this short answer is good. But I'm not sure if I would use this method for an Olympics. I think logarithms are simpler than the answer And have a good axiom to condense those complicated answers.❤

    • @lophocthienuc7345
      @lophocthienuc7345 Před 7 měsíci +2

      I dont think so. 2^m 65 why m

    • @gheorgheneacsu3356
      @gheorgheneacsu3356 Před 4 měsíci +2

      Me too! 😊😊

    • @chrissyday67
      @chrissyday67 Před 2 měsíci +2

      exactly! if students are good enough to do olympiad problems then they'd know powers of 3, 3, 9, 27, 81, 243 at least and also 2 to even higher powrers, 2, 4 , 8, 16, 32 etc so very easy to work out in less than 20 seconds . However I'm used to doing mental arithmatic as I never had a calculater when I was in school

  • @rubikaz
    @rubikaz Před rokem +94

    There is a problem here, when you write (x+y)(x-y)=5x13 you can not deduce that x+y=13 because you do not know if x+y is a natural number. If m is an odd number, then x=3^(m/2) and y=2^(m/2) are not natural numbers. So you have prove that if m is an even number, then m=4. it is very easy to check that there is only 1 solution.

    • @onlineMathsTV
      @onlineMathsTV  Před rokem +8

      Noted.

    • @nicadi2005
      @nicadi2005 Před 6 měsíci +3

      @rubikaz "So you have prove that if m is an even number" - Or, you can assume that m is even (thus making the quantities x+y and x-y (positive) integers etc.) and see whether it pays off - which it does, actually.
      "it is very easy to check that there is only 1 solution." - Indeed. The uniqueness of the solution is a direct consequence of the properties of the exponential function at work here...

    • @babetesfaye1001
      @babetesfaye1001 Před 4 měsíci +3

      ❤❤❤
      May I use any number Which have same numerator band denominator ? like 3/3 , 4/4 , 5/5....

    • @with.love.from.siberia
      @with.love.from.siberia Před 4 měsíci +1

      ​@@babetesfaye1001да, потому что оно равно 1

    • @Quasar900
      @Quasar900 Před 4 měsíci +5

      @@babetesfaye1001 the function f(x) = 3^x - 2^x where x>0 is strictly growing, therefore with x=6 , f(6) > 65 so x must be less than 6, and so on trying integers until finding x=4 or m=4

  • @nicholastergech8525
    @nicholastergech8525 Před 10 měsíci +134

    Similarly you can as well re-write 65 as 81-16....From there you make the bases of the two numbers to be similar with what you have on the left hand side..From there you take one of the corresponding bases and equate them together,when bases are the same powers will also be the same.

    • @usmanmusa8028
      @usmanmusa8028 Před 10 měsíci +2

      This is what I actually expected from him

    • @TomJones-tx7pb
      @TomJones-tx7pb Před 10 měsíci +3

      If you do not notice that 16 = 2**4, you are not a computer geek.

    • @lgmoses3876
      @lgmoses3876 Před 10 měsíci +3

      I did it,in my braine.

    • @sfqamd
      @sfqamd Před 10 měsíci

      czcams.com/video/sIlRHg730CA/video.htmlsi=ujk2KD_xjoMGqiP5

    • @shivaprasadmallikarjunaiah3751
      @shivaprasadmallikarjunaiah3751 Před 10 měsíci +19

      you are not mathematically solving the problem, but doing so by trial and error. These were smaller numbers so it is easy for anyone to come to that conclusion ( becausethe solution is "visible" in the numbers in front of you). In other words, how would you solve the same problem with entirely different and larger numbers involved? ...say 2059 for instance.

  • @alfredomulleretxeberria4239
    @alfredomulleretxeberria4239 Před 8 měsíci +63

    I was impressed by the analytic demonstration used to figure out that m = 4. Sometimes procedures can be more interesting to follow along than just knowing the result.

    • @italixgaming915
      @italixgaming915 Před 8 měsíci +5

      Well, you'll be impressed to see that the proof is not valid. If you suppose that m is an even number then x+y and x-y are integers but if it's an odd number then they are IRRATIONALS and you can't use 65=5*13.
      Plus the fact that even in the case where x+y and x-y are integers, 5*13 is not the only way to get 65, you must also look at 1*65...

    • @jarikosonen4079
      @jarikosonen4079 Před 5 měsíci

      ​​​​​@@italixgaming915This looks like diophantine method used here, which would work only with the integers.
      In case the solution works like in above case it could prove that no other integer solutions exist... But it seems 65x1 was not checked.
      This could work for 3^m-2^n cases also. Variables 'm' and also 'n' are often used for integers, but not necessarily always.

    • @claudiohagra
      @claudiohagra Před 4 měsíci +1

      You are the top!😊 You are a great teacher!!!!

    • @elmehdiazzouz7888
      @elmehdiazzouz7888 Před 3 měsíci +2

      the proof is not valid, we need to proceed much more cautiously with analytic ways, i d say : some more conditions/discussions needed to be added to the video .... i gree with italixgaming
      the arithmetic way stays safer ..

  • @samuelmayna
    @samuelmayna Před 8 měsíci +34

    You can also factor 65 into 65 and 1. This gives values of a and b as 33 and 32 hence b=2^(m/2)=32, m= 10 but m will have different value for 3^(m/2)= 33. Using logs(or ln) m=(log 33/log 3)=6.365

    • @eliasgitau7353
      @eliasgitau7353 Před 7 měsíci +2

      This makes sense and it is mathematically correct

    • @nicadi2005
      @nicadi2005 Před 7 měsíci +2

      @samuelmayna "You can also factor 65 into 65 and 1." - Yes, you can, but these won't be proper factors for 65, in the sense that ANY NUMBER could be "factored" as itself and one... Also, as you've seen yourself, this breaks the consistency of the original equation by forcing the unknown to take different values simultaneously - which is obviously not possible.

    • @user-uy7uu1rm5u
      @user-uy7uu1rm5u Před 6 měsíci

      m is an integer

    • @samuelmayna
      @samuelmayna Před 6 měsíci +4

      ​@@nicadi2005 but it is mathematically logical.Mathematics is about thinking all cases.

    • @samuelmayna
      @samuelmayna Před 6 měsíci

      ​@@user-uy7uu1rm5u but my approach is sound which shows that 65 and 1 won't work but it can give different answers for some equations.

  • @sanmus100
    @sanmus100 Před rokem +707

    How can one just assume that x+y = 13, and x-y = 5, respectfully, as there's 65 and 1 as well. Furthermore, this wouldn't really work if 65 had many more factors, making it more complicated, opening up to a lot more possibilities.

    • @onlineMathsTV
      @onlineMathsTV  Před rokem +179

      Nice question @Santhosh John. There are principles and rules that govern the operations in mathematics as it is for all other sphere of life.
      Once you are a maths student or tutor you must get urself familiarized with these rules.
      They become part of you and you know what to do once you have a math challenge/problem before you.
      Once a mathematician sees a math problem, his head automatically runs through different means of approaching the problem for a better solution.

    • @thegreathussar9442
      @thegreathussar9442 Před rokem +35

      When taking difference of squares if smaller part is 1(assuming it is the smaller part and it is) , m=2 and positive part becomes 5, not 65 therefore the result will be 5, not 65.
      If it has more factors, you just have to make arithmetical inferences and simplify it. That's it.

    • @naharmath
      @naharmath Před rokem +97

      3^(m/2) is not necesserly an integer!

    • @thegreathussar9442
      @thegreathussar9442 Před rokem +19

      @@naharmath but there is no other solution since both parts are exponential and even if m is a rational number the result wouldn't be integer( they are different primes). So this equation requires to be analyzed numerically first

    • @mariosantangelo9929
      @mariosantangelo9929 Před rokem +23

      Il professore ti ha risposto in maniera adeguata. Però io consiglio al professore di spiegare certe regole anche se ciò richiede qualche minuto in più. I fruitori di you tube non sono tutti matematici, ma persone desiderose di capire ed imparare.

  • @georiashang1120
    @georiashang1120 Před 9 měsíci +7

    65=13•5=(9+4)×(9-4)=9^2-4^2=3^4-2^4
    4 is the m;
    My high school math teacher used to tell me,the easiest way to understand an equation is to make them look the same,that is to say,we should make the brief side more complicated other than simplifing the complicated side for the most of the time.

  • @Amy-601
    @Amy-601 Před 8 měsíci +11

    The way I see it, 3 cubed is 27, less than 65, and 3 raised to 4 is 81. Therefore 65 is between 27 and 81. Upper bound, lower bound or range even. Now 81 would mean m is 4. So 2 should be raised to 4 also which gives us 16. 81 minus ➖ 16 is 65. So m is 4. The other ways are using log or binomial series which is overkill for smaller numbers. - Amy

  • @Sapped6
    @Sapped6 Před 8 měsíci +9

    A matemática é uma língua universal como a música. Parabéns, ótima técnica ❤

  • @user-oi3iv7oo4z
    @user-oi3iv7oo4z Před rokem +51

    It doesn't work if m is odd. In this case (x+y) and (x-y) aren't integer and can't be assumed as 5x13.

    • @danielrivera2278
      @danielrivera2278 Před rokem +6

      If you do the analysis, and supposing m is an integer, you can conclude m is even.
      That's because 3^m-2^m must be congruent to 0 mod (5). If m is even you get 1 mod 5 or 4 mod 5. But m being even, you have 0 mod 5 always.
      Anyways, it's not proven in the video, maybe it would be amazing to have hows and whys in the video

    • @user-oi3iv7oo4z
      @user-oi3iv7oo4z Před rokem +2

      @@danielrivera2278 of cause. I mean that "trick" in the video is not universal.

    • @Change_Verification
      @Change_Verification Před rokem +4

      @@danielrivera2278 and who even said that m must be an integer ?

    • @danielrivera2278
      @danielrivera2278 Před rokem

      @@Change_Verification exactly. I also think like that, that's because assuming integer was the first thing I said

    • @Eismann1
      @Eismann1 Před 9 měsíci

      Yes, he didn't do the preliminary work. But this is an important step to make tricks like this one work.

  • @Shirlippe
    @Shirlippe Před rokem +173

    Hi there! Here is an alternative solution. The difference of the powers on the rhs increases with m, and it is already greater than 65 for m=5. So m must be less than 5. Assuming m is an integer, and noticing that m=3 doesn't work, the only possible solution is 4.

    • @onlineMathsTV
      @onlineMathsTV  Před rokem +21

      Ya boss. You are very very correct and I like you approach sir.
      This is an indication that you are a master in this area.
      Respect sir...👍👍👍

    • @ronitmahawar1193
      @ronitmahawar1193 Před rokem +22

      but this only works if m is integer
      which we dont know it would be

    • @romanyukvictor
      @romanyukvictor Před rokem +10

      @@ronitmahawar1193 Exactly! To solve this you have to prove that the equation has single solution. Just shift 2^m to the right side and divide the entire equation on 2^m. As a result you will get the increasing function of m on the left (3/2)^m and decreasing function on the right (1+65/2^m) which can intersect only once. So there is only one root. The only way to find the root is to enumerate the integers and find out m=4.

    • @ifomichev
      @ifomichev Před rokem +10

      @@ronitmahawar1193 the solution proposed by the author of the video also works only for integers, because it relies on factorization of 65

    • @jccamargo99
      @jccamargo99 Před rokem +3

      For this reason many people don't like math.

  • @user-qo6ni5sm5p
    @user-qo6ni5sm5p Před 9 měsíci

    Solução genial. Parabéns. Muy interesante, hoy aprendí un buen método, gracias.

  • @GautamKumar-wx3sm
    @GautamKumar-wx3sm Před 5 měsíci

    Couldn't have thought of this approach. Thanks for this.

  • @elio9008
    @elio9008 Před rokem +46

    If you guess the solution, m=4 and present the equality as 3^m - 81 = 2^m - 16 , then it would be easy to prove that both functions (on the right and on the left) increase and therefore their graphs have only one intersection.

    • @ca1498
      @ca1498 Před 10 měsíci +2

      You need more than that. Two increasing functions can intertwine and cross each other all the time. But if they have one intersection, and after that one of them grows faster than the other all the time, then it follows that they won't intersect again. It's like two cars racing. They both increase their distance from the start all the time, but they could swap places many times during the race--unless one of them is always faster than the other after the point in which they were even with each other.

    • @ca1498
      @ca1498 Před 10 měsíci

      @@reginaldocalvo4361Each of the two sides can be a function e.g. y = 3^x - 81. The solution of the equation 'side 1' = 'side 2' is a number x (or m) where the two functions give the same y for the same x. You first find, by guessing, one x (or m) for which the two functions have the same y, which would mean that the particular m is one solution to the equation of function 1 = function 2 for some x. You then show that each of the functions only grows, and that the difference between the two y-s for each x (which is a function of x as well) also only grows. Therefore there can be only one value of x for which the difference is 0, so only one m (the one we already guessed) is the solution to the equation where one of the functions has the same y as the other for a given x.
      If I were participating in this Olympiad and solving this problem, I would have guessed 4, and then I would have argued that the first side 3^m... grows much faster than the other side 2^m for each subsequent m. I wouldn't be using derivatives, as I did not know any calculus in high school. But if they ask about integer solutions, I would talk about growth of y with respect to changes in m by +1. And if they did not limit it to whole numbers, I would probably still try to talk about slope of the graphs of the functions and hope to make an acceptable argument, as I don't see how you can analyze these functions without using calculus.

    • @user-xl3mg3om7s
      @user-xl3mg3om7s Před 9 měsíci

      @@reginaldocalvo4361
      3^m-81= f(m) this can be considered as a function depending on m
      2^m-16= g(m) also could be considered as function of m.

    • @user-xl3mg3om7s
      @user-xl3mg3om7s Před 9 měsíci

      @@ca1498 you are right, but
      When you see this function it is easy to notice that the growing path or direction is already know but partially

    • @Vitzyk
      @Vitzyk Před 9 měsíci

      Consequence is false.Contra-example x and x^3. Both increase but have 2 intersections

  • @therichcircle.8819
    @therichcircle.8819 Před rokem +8

    You tried here tutor Jakes. I have learnt something here. Just keeping on running this channel. More grace, love from Port Harcourt ❤.

    • @onlineMathsTV
      @onlineMathsTV  Před rokem

      It is our pleasure to serve you sir. Thanks for watching

  • @teamredx1pro952
    @teamredx1pro952 Před 3 měsíci

    Thank you for reminding me that you can't believe everything on the internet. Props to you i almost believed it untill i tested it with calculator. BRAVO YOU SHOULD WIN AN OSCAR

  • @fisicamatematicasprofewilliam
    @fisicamatematicasprofewilliam Před 5 měsíci +3

    profe en olimpiadas de las matematicas se aprende mucho. y con usted bastante. felicitaciones

    • @onlineMathsTV
      @onlineMathsTV  Před 5 měsíci +1

      Thanks a million sir, we appreciate this comment my good friend.

  • @user-pd7js7cy9m
    @user-pd7js7cy9m Před rokem +22

    It can decide differently. The function (1) f(x)=3^x-2^x -is increasing . { for x>0 (2) 3^x>2^x ; x10 . (3) f(x2)-f(x1)= ……..=3^x1*[3^(x2-x1)-1]-2^x1*[2^(x2-x1)-1 ]> 2^x1*[2^(x2-x1) -1 ]-[ “--“ ]=0 ; (3) f(x2)-f(x1)>0 !!!!! So , it takes all its meanings once a time. f(4)=3^4-2^4=65 . So , x=4 - is the only root of equation ! Respectfully , Lidiy

    • @onlineMathsTV
      @onlineMathsTV  Před rokem +3

      The elders in our mist are highly learned.
      Love your detailed explanation sir.
      Thanks for finding our time to watch our content and commenting even at this age of yours sir.
      Much respect and we All @onlinemathstv love you dearly...💖💖💕💕

    • @Lernen-mit-Rudi
      @Lernen-mit-Rudi Před rokem

      There is no X! 😂but however, good job!

    • @andreasandre4756
      @andreasandre4756 Před 11 měsíci +1

      Pay attention that m=constant not variable, so M must be grater then 1 otherwise solution will not be true because ln1=0 or 3-2=1 which is not equal 65.
      So M>1 and could be anything. So if it is not an integer number? So if it is not equal 65 but 63.5 - ?

    • @ADSemenov_ru
      @ADSemenov_ru Před 10 měsíci +1

      You took the words right out of my mouth. :)

    • @Marat7973
      @Marat7973 Před 10 měsíci

      Здравствуйте,Лидий! Не ожидал Вас здесь увидеть)

  • @johnpalagye7036
    @johnpalagye7036 Před rokem +15

    Love your video! I got lost half way and I did not know that you could just square the exponents and make them equal

    • @onlineMathsTV
      @onlineMathsTV  Před rokem +3

      We are glad you love what is happening here.
      We promise to give more educative contents in the area of mathematics with the help of God.
      Love you.....💖💖💕💕

    • @karlm9584
      @karlm9584 Před 7 měsíci

      I like this too. I have done it with roots before. Roots are just indices but I wouldn't have thought of doing it.

  • @dannychown2593
    @dannychown2593 Před měsícem

    Your work opens up the horizon of my mind how to approach this type of question. Thank you.

  • @marcio5457
    @marcio5457 Před 9 měsíci

    Solução genial. Parabéns

  • @airtonreis2675
    @airtonreis2675 Před 11 měsíci +33

    Parabéns. Não sei falar nada em inglês e mesmo assim consegui aprender com sua aula. Até eu estou surpreso de ter assistido sua aula até o final sem saber se iria entender o seu modo de esnsinar. A matemática pode ser universal, mas o jeito de ensinar é fundamental.

    • @onlineMathsTV
      @onlineMathsTV  Před 11 měsíci +12

      @Airtonreis, we want to sincerely say you are such a wonderful person and thanks a million for watching our contents despite the language barrier.
      We all @OnlineMathstv deeply cherish and love you from the depth of our hearts sir. ❤️❤️❤️💖💖💖💕💕💕🙋🙋🙋

    • @airtonreis2675
      @airtonreis2675 Před 11 měsíci +1

      @@onlineMathsTV 🤝

    • @MATHSHEADMASTER
      @MATHSHEADMASTER Před 9 měsíci +1

      You are welcome to subscribe to our channel for more interesting math problems.

    • @pedrooo13
      @pedrooo13 Před 5 měsíci

      Brasil tá em todo lugar não tem jeitoooooo

  • @RoderickEtheria
    @RoderickEtheria Před rokem +18

    Solved 3^m-2^m=65 by just thinking about the first whole number power of 3 above 65.

    • @dilphek
      @dilphek Před rokem

      It is not about finding it. Olympiad is a school competition teaching kids to solve these problems mathematically

    • @bleh-zj1hy
      @bleh-zj1hy Před 8 měsíci

      ​​@@dilphekyou guys got a different Olympiad or something? Here Olympiads (for the kids, totally different type from the subjective ones) are mcqs and the subjective ones are like 3 questions in 3 hrs and if you're able to do even 1 you're qualified (you can imagine the toughness so it's really not for the kids)

  • @jamesclerkmaxwell8545
    @jamesclerkmaxwell8545 Před 9 měsíci +1

    Parabéns. Resolução muito boa.

  • @user-rz7ym5ts1e
    @user-rz7ym5ts1e Před 9 měsíci +1

    I love how the way you work out the question..❤❤

  • @jaimeduncan6167
    @jaimeduncan6167 Před rokem +23

    Another option: the solution is pretty clear, it's a small number one can calculate with the mind. Then it reduces to show that their solution is unique. One can use calculus to show that the analog continuos function is monotone for x>4 and be done with it, or use induction to show that it grows on the integers for n>4.

    • @onlineMathsTV
      @onlineMathsTV  Před rokem +4

      Your approach is superb sir. I find it very fascinating and I will try it out in subsequent math challenges.
      Thanks for sharing this nice and wonderful procedure with OnlinemathsTV.
      You the boss and much respect boss...👍👍👍

    • @GirGir183
      @GirGir183 Před 9 měsíci

      I think he used a simplified equation for the demonstration. When it's not so simple and the numbers are much larger, then this method can be used as well.

  • @ravirajshelar250
    @ravirajshelar250 Před 9 měsíci +5

    I did it in a different way.
    Write 3 as 2+1 and perform binomial expansion, so the 2^m cancels and subtract one from both sides. We have 64 on one side and some series on other side.
    Notice that the series 2 + 2^2 +.....+ 2^m will definitely be smaller than series on left which is equal to 64.
    So make this G.P. sum less than 64, you will get that m should be less than 5, and once you have known this, you have proved that you just need to find a solutions less than 5 and those will be the only solutions.
    Only m=4 works out.

    • @albajasadur2694
      @albajasadur2694 Před 8 měsíci

      a good method to find the range of m and it makes sensible checking easier with limited number of m

    • @theboss73104
      @theboss73104 Před 2 měsíci

      Yeah

  • @karlm9584
    @karlm9584 Před 7 měsíci +2

    I like how you used indices to get past that first section. I've never really been good at spotting where to use substitution, the section when you brought in x and y

    • @onlineMathsTV
      @onlineMathsTV  Před 7 měsíci

      Hahahaha....thanks a bunch my good friend and thanks for watching our contents consistently.
      We all here love deeply....💖💖💕💕😍😍

  • @brunodelenclos6035
    @brunodelenclos6035 Před 4 dny

    Belle démonstration.Merci beaucoup..

  • @franciscodeassisbrandaobra898

    exercicio maravilhoso🥰🥰❤❤❤❤❤❤

    • @onlineMathsTV
      @onlineMathsTV  Před rokem +1

      Thanks a millions sir, we love you ❤️❤️💖💖💕💕😍😍

  • @Psykolord1989
    @Psykolord1989 Před rokem +12

    Before watching:
    Alright, so, we are looking at exponential functions. 3^m - 2^m = 65.
    First, we can rule out m=1 and anything below; the difference between those would be smaller than between 3 and 2, and thus much lower than 65.
    Next, notice that we are dealing with an integer on the right. This heavily implies (but *does not necessarily guarantee* ) that 3^m and 2^m are both integers as well. If both are integers, then m must also be an integer. So we should start with integers. (If we were dealing with a mixed number instead, this would be much more complex; as it stands, we can just plug in integers and see which one works).
    Let's start with the first x that gives us 3^m >65, namely 4. 3^4 = 9^2 = 81, and 2^4 = 4^2 = 16
    So for m= 4, we have 81-16 = 65. Fortunately for us, this checks out, and thus we have our answer of *m = 4*

    • @onlineMathsTV
      @onlineMathsTV  Před rokem

      Thanks for this thorough explanation. You the best. 👍👍

    • @RoderickEtheria
      @RoderickEtheria Před rokem

      Negative m gets fractions.

    • @Psykolord1989
      @Psykolord1989 Před rokem

      @@RoderickEtheria Yes, you are correct, and I see I made a typo in the "so for m=4 we have..." section by putting a negative in front of the 4. Fixed now.
      I don't imagine it was a huge problem since in the section right above it,and at the very end of that section, I used 4, but it still may have confused some people.

  • @danielmelo5609
    @danielmelo5609 Před 8 měsíci +2

    Excellent explanation. It seemed very complicated, but it turned out to be easier than expected. A hug

  • @user-og4vc7ey2y
    @user-og4vc7ey2y Před 4 měsíci

    I'm impressed with this explicit methodology 😊(12:16am)

  • @umeshkhetan
    @umeshkhetan Před rokem +11

    If a.b=65, a and b can have infinite values. So, the tutor has just one answer where multiple answers are possible.

    • @onlineMathsTV
      @onlineMathsTV  Před rokem +2

      Yes it has multiple answer but for the sake of this tutorial we restricted ourselves to this solution sir.
      Thanks for this observation.
      Much love....💕💕👍👍

    • @mustaphaolunrebi8100
      @mustaphaolunrebi8100 Před 9 měsíci

      I think you add that, where m is an integer. It makes it complete. The only integer factors 65 has are 5 and 13. Nice solution 🎉

    • @ivandonchev474
      @ivandonchev474 Před 9 měsíci

      Sorry but this video is the most useless shit I have seen. You solved it by guessing and overcomplicated massively

    • @speedsterh
      @speedsterh Před 8 měsíci

      @@mustaphaolunrebi8100 No, 65 has 4 factors: 1, 5, 13, 65.
      The equation with 2 other factors should be explored for completeness

  • @mdmahin7299
    @mdmahin7299 Před rokem +9

    Very well solution but the rules are very lengthy or Expensive ☺️
    So if we assume the value of "m" here from 1-4
    We are easily getting the value of m
    Such as
    Let , m=1,2,3,4,... ♾️
    and now,
    3^1-2^1=1≠65
    again
    3^2-2^2=5≠65
    And now if we let, m=4 then
    3^4-2^4=65=65
    So we can easily get m=4😊

  • @Saibatou
    @Saibatou Před 7 měsíci

    You are just great! A very good approach.

  • @KidusYared-ox6qb
    @KidusYared-ox6qb Před 9 měsíci +1

    Thank you so much. That's the easy way to solve it❤👍

  • @owlsschoolofmath9732
    @owlsschoolofmath9732 Před rokem +2

    Great! Its a fun problem. I did something kind of similar with difference of 2 squares.

  • @Quasar900
    @Quasar900 Před 10 měsíci +5

    the function f(x) = 3^x - 2^x where x>0 is strictly growing, there for if x=6 f(x) > 65 so x must be

    • @onbored9627
      @onbored9627 Před 5 měsíci +1

      All I did was think whats the first interger power of 3 that goes past 65, 3 is 27 so the answer is 4. Then just test. and it gave right answer. simple.

    • @Quasar900
      @Quasar900 Před 5 měsíci

      @@onbored9627 Where are you from ? Cause how did you know I was still alive after 4 months ? 🙂Here is a secret about me : I 've never studied Mathematics in English !

    • @onbored9627
      @onbored9627 Před 5 měsíci

      @@Quasar900 Ah, I apologize I only speak English. I'm from the USA. I wasn't trying to take away from your explanation I should've been more clear on that, I just thought yours was so good I didn't even need to say. Figuring out it's strictly growing is clever as hell. I didn't even think of that. I meant simple, as in, my answer was simply a guess really and it worked.

    • @Quasar900
      @Quasar900 Před 5 měsíci

      @@onbored9627 Oh please Sir , no need to Apologise , The fact that I've never studied math in English doesn't mean I don't know English 🙂 It's just I'm not that familar with English terms in math !
      But Thank God I do speak and read : French, English, Arabic, Spanish + some Japanese !
      I did study math in French (after high school) and Arabic (until high school) , but that waaaas 21 years ago !
      What class are you in ? I hope you're safe from those ongoing blizzard storms !
      Greetings From Morocco and Free Palestine 🙂

    • @Quasar900
      @Quasar900 Před 5 měsíci

      @@onbored9627 I think you do know these techniques involving the continuity of a function, the intermediate values etc.. to solve equations !
      For example :
      solving in IR set :
      Arctan(x+1)+Arctan(x-1)=Pi/4

  • @chrisw4562
    @chrisw4562 Před 9 měsíci +2

    Nice job! I got the answer by trial and error, but this way of getting a deterministic solution is really cool.

    • @italixgaming915
      @italixgaming915 Před 8 měsíci

      Actually this proof is wrong. If m is an even number then x+y and x-y are integers but if m is an even number then they are IRRATIONALS and the unicity of the prime factors works only with integers. Euler did the same kind of mistake when he tried to demonstrate the Fermat theorem (that time he used complex numbers).

  • @ludmilak9396
    @ludmilak9396 Před 2 měsíci

    Очень грамотное изложение, и очень удобно следить на доске. Однозначно плюс!🎉

  • @pwmiles56
    @pwmiles56 Před rokem +6

    In a slightly fancier approach we can make a recursion
    2(3^m - 2^m) + 3^m = (2+1)3^m - 2^(m+1)
    = 3^(m+1) - 2^(m+1)
    Put a(m) = 3^m - 2^m
    a1 = 3^1 - 2^1 = 1
    a2 = 2 a1 + 3^1 = 5
    a3 = 2 a2 + 3^2 = 19
    a4 = 2 a3 + 3^3 = 65, done

    • @onlineMathsTV
      @onlineMathsTV  Před rokem +4

      Wow!!! This approach is impressive but a bit obscure sir.

    • @vejayashanker
      @vejayashanker Před 9 měsíci

      can u explain what u mean by obscure pls😂

    • @soltanchalkarow905
      @soltanchalkarow905 Před 6 měsíci

      can you help me sir?
      if ab+bc+ca=1
      prove:
      sqrt(a + (1/a)) + sqrt(b + (1/b)) + sqrt(c + (1/c)) >= 2( sqrt(a) +sqrt(b) + sqrt(c) )

  • @DrMikeE100
    @DrMikeE100 Před 9 měsíci +4

    It took a fraction of a minute to recognize m = 4, but as a mathematician, I did find this interesting. (Dr. Mike Ecker)

    • @JaroGoraJ
      @JaroGoraJ Před 9 měsíci

      I think because it was easy question

  • @sevgininbahcesi5222
    @sevgininbahcesi5222 Před 4 měsíci

    Perfect.Thank you sir.

  • @CleopatraNadesan-kn3jg
    @CleopatraNadesan-kn3jg Před 9 měsíci

    Excellent tutorial. Thank you very much

  • @FractAlkemist
    @FractAlkemist Před rokem +37

    I have been learning Genetic Algorithms in Python; they are good for problems like this.
    The value I get for 'm' is 3.97, 3.99, 4.00, etc.; different each time as there is a random element for the convergence.
    A little intuition is also required;
    If you plug 4.0 into the equation you get correct 65. Program run time ~5 seconds.

    • @onlineMathsTV
      @onlineMathsTV  Před rokem +3

      Wow!!! Nice sir.

    • @pierrecurie
      @pierrecurie Před rokem +4

      If you're going the numerical route, bisection search is much faster.

    • @mistertwister1015
      @mistertwister1015 Před rokem

      Blunt enumeration of roots is not always the best solution)

    • @victorfildshtein
      @victorfildshtein Před rokem +1

      Hello. I program in PureBasic. I made this task by binary search method, 30 iterations,
      result 3.9999999991, precision 0.0000000047. Time is almost instantaneous.

    • @wilsonuche9389
      @wilsonuche9389 Před 10 měsíci

      You need to review the Python solutions cos only 4 is an exsct solution. 3.97 is far from it, 3.99 is just an approximate

  • @mathadict1
    @mathadict1 Před 10 měsíci +3

    Keep up the great work man happy to see somoen in our beloved African continent devoting a portion of their time into this much love from Morocco 🇲🇦 MA

    • @onlineMathsTV
      @onlineMathsTV  Před 10 měsíci

      Much appreciated sir.
      Thanks for watching our contents and the encouragement sir.
      Much love from everyone @OnlinemathsTV to you sir 💖💖❤️❤️💕💕💕

    • @Quasar900
      @Quasar900 Před 10 měsíci

      @@onlineMathsTV
      the function f(x) = 3^x - 2^x where x>0 is strictly growing, there for if x=6 f(x) > 65 so x must be

    • @Quasar900
      @Quasar900 Před 10 měsíci

      the function f(x) = 3^x - 2^x where x>0 is strictly growing, there for if x=6 f(x) > 65 so x must be

  • @zulfqarali2994
    @zulfqarali2994 Před 2 měsíci

    Gentleman I appreciate your work.

  • @jamesmichuki5804
    @jamesmichuki5804 Před 6 měsíci +1

    Thanks teacher. Today I've learnt something very new in maths. Am really surprised....

  • @Toxa_Azimov
    @Toxa_Azimov Před rokem +11

    Слева возрастающая функция при m>0 ( можно найти производную и убедиться ), справа постоянная функция, значит у них может существовать только одна точка пересечения, методом оценки m=4

    • @onlineMathsTV
      @onlineMathsTV  Před rokem

      Bravo 👍👍👍
      You the best sir.

    • @onlineMathsTV
      @onlineMathsTV  Před rokem +1

      Bravo 👍👍👍
      You the best sir.

    • @PlumbuM871
      @PlumbuM871 Před 6 měsíci

      Чисто случайно подставил вместо m 4, и всё сошлось! Везёт мне

  • @AndrewUnruh
    @AndrewUnruh Před 9 měsíci +5

    Clever solution! One thing I did not get is how you knew that m was a positive integer - or was this just an assumption that happened to work? For example, if the problem had been 4^m - 3^m = 65, m would be approximately 3.36.

    • @charleskaruru481
      @charleskaruru481 Před 8 měsíci

      m can never be a negative otherwise we wont have 65but fraction

  • @user-my1vs2ep5c
    @user-my1vs2ep5c Před 14 dny +1

    Thank you very much!
    Без перевода мне всë так понятно было👍

  • @gabrielschiteanu4963
    @gabrielschiteanu4963 Před 2 měsíci +2

    Rewrite the equation as 3^x = 65 + 2^x. We can safely divide by 3^x and then we have that 1 = 65 * (1/3)^x + (2/3)^x. Because the function on the R.H.S. is strictly decreasing, being the sum of 2 other strictly decreasing functions, it means that f(x) = 1 has one solution at max. We notice that x=4 checks, so that is our only solution.

    • @onlineMathsTV
      @onlineMathsTV  Před měsícem +1

      Bravo!!!
      You the best sir.
      Maximum respect sir 🙏🙏🙏

  • @michaelsidorov5508
    @michaelsidorov5508 Před rokem +5

    Остроумно и красиво! Как и вся математика.

    • @onlineMathsTV
      @onlineMathsTV  Před rokem +1

      Thanks for this wonderful comment sir.
      Love you....💕💕

    • @user-nv9rw7nh5w
      @user-nv9rw7nh5w Před rokem

      Очень, очень. Надо же как можно. Удивительно!

  • @user-qt9qo3ez9x
    @user-qt9qo3ez9x Před 10 měsíci +14

    Сначала поделить обе части на 2^m. Тогда слева будет возрастающая функция, справа убывающая. Тогда уравнение имеет не более одного корня. Подобрать корень не сложно. 9 класс, ничего сложного. За проведенное решение минус: нигде не доказано отсутствие других решений, переход к системе ничего не обосновывает.

  • @michaellockett4044
    @michaellockett4044 Před 2 měsíci

    Difference of squares into u-substitution. Excellent methodolgy.

  • @xuanhungnguyen7667
    @xuanhungnguyen7667 Před 3 měsíci

    Chính xác.anh biến đổi 65=5×13.đây mới là mấu chốt của bài.và tư duy hàm số mũ...tuyệt.

  • @88kgs
    @88kgs Před rokem +12

    Sir, we can also do this by hit and trial method, assuming different values for m=1,2,3,4....
    But your way was also very nice 👍👍.
    Thank you for this video sir🙏

    • @onlineMathsTV
      @onlineMathsTV  Před rokem +2

      Yes, but that will only work if the working process is not in the examination but this is needed when the examiner wants you to show your procedure step by step...👍👍👍

    • @gregfarnham5651
      @gregfarnham5651 Před rokem +1

      Yes, trial and error could work if we assume m is a positive integer. I don't believe that was a given, however.

    • @ralfimuller8948
      @ralfimuller8948 Před rokem +1

      @@gregfarnham5651 The solution in the video also made use of the assumption that m is an integer. Otherwise the factorization of 65 would not be unique. Actually, the hit and trial method should be entirely ok.

    • @gregfarnham5651
      @gregfarnham5651 Před rokem +1

      @@ralfimuller8948 Agree. Thank you.

    • @danielrivera2278
      @danielrivera2278 Před rokem +1

      Also, by trial and error you can't prove that's the obly answer

  • @AntoninaKa-es8tv
    @AntoninaKa-es8tv Před rokem +5

    Очень интересное решение! Спасибо!

    • @onlineMathsTV
      @onlineMathsTV  Před rokem +2

      You are welcome always even as we look forward to seeking better ways of serving you better in our services to you on this channel.
      We at Onlinemathstv love you without reservation sir....💕💕💕

  • @marcofidelpenaavila2129
    @marcofidelpenaavila2129 Před 4 měsíci

    Extraordinario , thanks so much.

  • @jwilson4163
    @jwilson4163 Před 8 měsíci

    Great lesson! Thanks. (From São Paulo/BR)😃

  • @paulmiddletonphotography4368

    Hello Online Maths TV,
    I really like your approach with this and how your deliver the proof. It is elegant and uses several skills. Your pace of delivery is really good too.
    As a suggestion for completeness in your proof, can you please include a determination table of which factors of 65 and their order are valid for consideration for equating to (x+y).(x-y). A table similar to;
    (x+y) . (x-y) | x | y | m from x=3^(m/2) | m from y=2^(m/2) |
    1 . 65
    65 . 1
    13 . 5
    5 . 13
    If you calculate x and y and then m from x=3^(m/2) and from y=2^(m/2) for each of these arrangements, only (x+y) . (x-y) = 13.5 provides valid and consistent values for m. So 3 out the 4 arrangements can be mathematically eliminated.
    This would verify that the only valid arrangement and values of the factors is 13.5.
    I hope this helps.
    Cheers,
    Paul.

    • @onlineMathsTV
      @onlineMathsTV  Před rokem +2

      What a wonderful guide @Paul Middleton. Very very helpful sir.
      Respect boss. ❤️❤️❤️👍👍👍

    • @onlineMathsTV
      @onlineMathsTV  Před rokem +3

      Am compared to drop another comment in respond to your first comment sir. Am speechless and overwhelmed by your systematic construction of this comment. It is so encouraging.
      This is because you appreciated my little effort in the mist of professors and further suggested a wonderful/a great approach to me on solving this same problem or similar problem in my subsequent videos. Sir, with all humility we are glad to meet with you and have you here.
      Much love sir....💖💖💖💕💕💕❤❤❤

    • @paulmiddletonphotography4368
      @paulmiddletonphotography4368 Před rokem

      @@onlineMathsTV You are very welcome indeed.

    • @paulmiddletonphotography4368
      @paulmiddletonphotography4368 Před rokem

      @@onlineMathsTV Thank you for your very kind words of appreciation. It certainly is my pleasure to help you and I am really pleased that my suggestion will assist you and into the future with different proofs. May you increase your subscribers, students and people interested in learning your very clear, easy to follow and forthright teaching of maths. All the best to you for the future.

  • @mathtv3982
    @mathtv3982 Před rokem +5

    First you must show that m is even positive integer

  • @bdh9202
    @bdh9202 Před 4 měsíci

    You worked hard, I found the result by giving the value of m to 4 in 10 seconds, but it is important how the solution is, but I still think it is too long. Greetings from Turkey.

  • @knguyen3348
    @knguyen3348 Před 5 měsíci

    You are the best. Thanks

  • @divonsirlopes5409
    @divonsirlopes5409 Před rokem +4

    There is a faster workaround, with the assumption that the number m is integer. The term on the right is less than the term on the left. For simplicity, we can assume that the term on the right is zero. This results in:
    3^m = 65.
    m is greater than 3, because 3^3 = 27.
    m can be 4 because 3^4 = 81.
    Let's test m = 4:
    3^4 - 2^4 = 65
    81 - 16 = 65
    65 = 65

    • @onlineMathsTV
      @onlineMathsTV  Před rokem +2

      Wow!!!
      This is fantastic. I love this approach sir. We have gained some values from this procedure sir. Thanks for dropping this sir.
      Respect.....👍👍👍
      Much love....💕💕💖💖❤️❤️

    • @divonsirlopes5409
      @divonsirlopes5409 Před 11 měsíci

      There is a faster workaround, with the assumption that the number m is integer. The term on the right is less than the term on the left. For simplicity, we can assume that the term on the right is zero. This results in:
      3^m = 65
      m = log(65)/log(3) = 3,8
      m is greater than 3,8
      Let's test m = 4
      3^4 - 2^4 = 65
      81 - 16 = 65
      65 = 65

    • @MONSTER2013
      @MONSTER2013 Před 9 měsíci +1

      Base on which level you’re at. This question and video are made for yr 10-11? So he gave the solution at that grade. Above yr 12 can use other tools as log/ ln skilfully to solve it.

    • @divonsirlopes5409
      @divonsirlopes5409 Před 9 měsíci

      Thanks for the info.

    • @foudilbenouci482
      @foudilbenouci482 Před 8 měsíci

      ^you found one solution doesn t mean you found all solutions

  • @Curufin1984
    @Curufin1984 Před rokem +9

    This is so overly complicated.. Just check low values of m and find that m = 4 works.. Then use a simple analysis tool to show uniqueness of the solution e.g. by showing that function f(m) = 3^m - 2^m is strictly increasing for m>=1.
    Additionally your solution contains errors and missing steps:
    1. If you do the trick with 3^(m/2)^2 - 2^(m/2)^2 = 65 then you presuppose that m is even, because if m is odd then 3^(m/2) is not natural anymore, so you cannot use the natural divisors of 65 in the following steps anymore.
    2. Even if that approach worked (because you somehow proved that m must be even): After you rewrite the equation as x^2 - y^2=65, then you have to consider *two* pairs of solutions 65 = 65 * 1 and 65 = 5 * 13.
    Long story short, lots of mistakes in your video unfortunately.

    • @onlineMathsTV
      @onlineMathsTV  Před rokem +1

      Thanks for this keen observation and I really appreciate this comment sir.
      Noted. I will do more detailed work on subsequent videos.
      You the best and much love for this detailed comment....💕💕💕

    • @ivandonchev474
      @ivandonchev474 Před 9 měsíci

      Yes this video is shit full of errors. The fact that it has so many errors and is hugely overcomplicated at the same time makes it complete and utter shit

  • @mikeblings1504
    @mikeblings1504 Před 4 měsíci

    As a math teacher, this is a plus to me. You are amazing 👏

  • @rajendrasinghbisht2628
    @rajendrasinghbisht2628 Před 3 měsíci

    Nice solution. It makes so easy, the way you explained.

  • @BrukFikru
    @BrukFikru Před 7 měsíci +1

    Nice one ...... good teacher ...... stay teaching stay doing more math tricks ...... i liked it

  • @Luis-zj4dv
    @Luis-zj4dv Před měsícem

    Genial, muchas gracias!

  • @marcelo372
    @marcelo372 Před 2 měsíci

    Excelente. Thank you.

  • @clownphabetstrongwoman7305
    @clownphabetstrongwoman7305 Před 4 měsíci

    Maybe it's a bad argument but I would say for a^m - b^m = X, and a, b, m belong to N, a^m> X > a^m-1.
    Here 3^m>65>3^m-1
    81>65>27 => m=4.

  • @dinlendiricidrtv
    @dinlendiricidrtv Před měsícem

    Thank you very much my dear friend

  • @abdussalammondol6234
    @abdussalammondol6234 Před 9 měsíci

    Many many thanks.❤️💚❤️💚

  • @composer7325
    @composer7325 Před 8 měsíci

    Excellent, thank you.

  • @tejendramohenbaisya8529
    @tejendramohenbaisya8529 Před 9 měsíci

    Problem solved in a very intelligent way.

  • @TheMightyOdin
    @TheMightyOdin Před 9 měsíci

    I did this in my head in less than a minute.
    3,9,27,81(bigger than 65)
    2,4,8,16( subtract 16 from 81)…. Works.

    • @daintydawn2508
      @daintydawn2508 Před 3 měsíci

      Yeah, it's a bit nd trial method. But if the value of m had been bigger then, it wouldn't work

  • @nnaammuuss
    @nnaammuuss Před 9 měsíci

    m≤2 makes it too small and don't work. So, m must be even, as 3^m ≡ 3 (mod 4) for m odd, but 65 ≡ 1 (mod 4). On the other hand, 3^m - 2^m = 3^{m-1} + ... ≥ 3^{m-1} and is strictly increasing. Since, 3⁴ = 81 is already too big, we must have m

  • @evasolovyova
    @evasolovyova Před 9 měsíci +2

    Отличное решение, всё ясно и понятно! Как хорошо, что математика у всех одна, математика сближает людей. Мы понимаем друг друга, говоря на разных языках!

  • @antoniogomesfigueiredo7835
    @antoniogomesfigueiredo7835 Před 9 měsíci

    Gostei muito da resolução dessa questão. Não entendi suas palavras, pois, sou português, todavia, a resolução da equação entendi perfeitamente. OK? Obrigado. Thank you very much.

  • @forgottenlegacy5929
    @forgottenlegacy5929 Před 2 měsíci

    Wonderfully explained, very informative.
    But it’s sometimes more convenient to use the easy method

  • @math_qz_2
    @math_qz_2 Před 6 měsíci +1

    Very instructive task

  • @georgesadler7830
    @georgesadler7830 Před 8 měsíci

    Thank you for the video.

  • @user-ne7pu8ib7y
    @user-ne7pu8ib7y Před 5 měsíci +1

    3ᵐ-2ᵐ=65
    y=3ˣ-2ˣ
    y=65
    если построить оба эти графика, то будет видно, что уравнение имеет единственное решение, поэтому можно попробовать подобрать корень подбором:
    х=1: 3¹-2¹=1; 1≠65
    х=2; 3²-2²=5; 5≠65
    х=3; 3³-2³=19; 19≠65
    х=4; 3⁴-2⁴=65; 65=65
    х=4 корень
    m=4

  • @user-tw4fp1jg8g
    @user-tw4fp1jg8g Před 5 měsíci

    i really learn a SOME TRICK from this thankss

  • @charleskaruru481
    @charleskaruru481 Před 8 měsíci +1

    in maths olympiad time is very critical...what is important is the answer...so the best way is to solve maths olympiad is to have very basic maths then the rest is analysing to get the pattern...so what i did is just look at indecies of 3 that have the last number such that if we subract an indecies of 2 and get 65

  • @user-zu9lw3zb4d
    @user-zu9lw3zb4d Před 7 měsíci +1

    Mathematics is all abt observation and understanding thee pattern thx a ton Buddy❤

    • @onlineMathsTV
      @onlineMathsTV  Před 7 měsíci

      Thanks a million for watching and dropping this wonderfully encouraging comment sir. The true is that mathematics is all about finding a solutions to problems/challenges. We really appreciate the fact that you watch and dropped this comment to clarify some debts in the minds of so many viewers and subscribers here.
      Much love from all of us @OnlinemathsTV....💕💕💕💖💖💖❤️❤️❤️

  • @elhassanidnarour672
    @elhassanidnarour672 Před 8 měsíci

    You must first determine if m is even, or do you also handle the second case of m being odd.

  • @najmulhussainlaskar7118

    I learnt something new from this video... And I wanna say something to dear sir please do you job not listen other criticism... Love you sor i am from India ( Bengali) i am also a Maths teacher 🧡🤍💚

  • @harshkumarthakur_
    @harshkumarthakur_ Před 2 měsíci

    Thank you sir for solve this type of problem

  • @ramswaroop1881
    @ramswaroop1881 Před 9 měsíci

    Nicely explain. Thanks

  • @mymathtutoreg2628
    @mymathtutoreg2628 Před 8 měsíci

    3^m>65 use log of both sides to solve the inequality we get m>3.8 so start to check m=4 in the original equation the equation is satisfied and m = 4

  • @mathurinenokou
    @mathurinenokou Před 9 měsíci

    Thanks you sir 🙏🙏

  • @houssamelayoubi4527
    @houssamelayoubi4527 Před 8 měsíci

    Perfect, thank you!

    • @onlineMathsTV
      @onlineMathsTV  Před 8 měsíci +1

      Thanks a million my good friend for watching and laving a comment behind to encourage us at what we do here.
      We deeply love you sir....💖💖💖

    • @houssamelayoubi4527
      @houssamelayoubi4527 Před 8 měsíci

      @@onlineMathsTV i am very happy to hear this from your side and appreciate it ❤

  • @funomathics4819
    @funomathics4819 Před 9 měsíci

    Awesome formation of solution,,,,,
    wahhhh!!!

  • @Winz_Breaker
    @Winz_Breaker Před 3 měsíci +1

    Thankyou you,i understand😊